- 3/5 + - 8/2 = in simplified way.

Answers

Answer 1

Answer:-23/5

Step-by-step explanation:

-3/5-8/2

-3/5-4

-3+20/5

-23/5

Answer 2

Answer:

-23/5

Step-by-step explanation:

-3/5 + - 8/2

Find the least common multiple of 5 and 2.

That would be 10.

That means we have to multiply the first fraction by 2/2 and the second one by 5/5

-3/5*2/2

Multiply the numerators together and the denominators together.

-6/10

Repeat with the second fraction.

-8/2*5/5

-40/10

Our new expression is

-6/10+-40/10

-46/10

Simplify

-23/5

As a mixed number that would be:

-4 3/5


Related Questions

evaluate the following -3 - (-8)

Answers

Answer:

The answer to the problem is 5.

Answer:

5

Step-by-step explanation:

[tex]-3-\left(-8\right)\\\\\mathrm{Apply\:rule}\:-\left(-a\right)=a\\=-3+8\\\\\mathrm{Add/Subtract\:the\:numbers:}\:\\-3+8\\=5[/tex]

What will be the remainder when 6x ^5+ 4x^4 -27x^
3
- 7x² + 27x + 3/2 is divided by (2x^2 - 3)
^2

Answers

Answer:

Remainder = (3145/8)x - 408

Step-by-step explanation:

We want to find the remainder when 6x^(5) + 4x⁴ - 27x³ - 7x² + 27x + 3/2 is divided by (2x² - 3)²

Let's expand (2x² - 3)² to give ;

(2x - 3)(2x - 3) = 4x² - 6x - 6x + 9 = 4x² - 12x + 9

So,we can divide now;

______________________

4x²-12x+9 |6x^(5)+4x⁴-27x³-7x²+27x+3/2

First of all, we'll divide the term with the highest power inside the long division symbol by the term with the highest power outside the division symbol. This will give;

3/2x³

______________________

4x²-12x+9 |6x^(5)+4x⁴-27x³-7x²+27x+3/2

6x^(5)-18x⁴-(27/2)x³

We now subtract the new multiplied term beneath the original one from the original one to get;

3/2x³

______________________

4x²-12x+9 |6x^(5)+4x⁴-27x³-7x²+27x+3/2

6x^(5)-18x⁴-(27/2)x³

¯¯¯¯¯¯¯¯¯¯¯¯¯¯¯¯¯¯¯¯¯¯¯

22x⁴+(27/2)x³+27x +3/2

We'll now divide the term in new polynomial gotten with the highest power by the term with the highest power outside the division symbol. This gives;

(3/2)x³ + (11/2)x²

______________________

4x²-12x+9 |6x^(5)+4x⁴-27x³-7x²+27x+3/2

6x^(5)-18x⁴-(27/2)x³

¯¯¯¯¯¯¯¯¯¯¯¯¯¯¯¯¯¯¯¯¯¯¯

22x⁴+(27/2)x³+27x +3/2

22x⁴-66x³ + (99/2)x²

¯¯¯¯¯¯¯¯¯¯¯¯¯¯¯¯¯¯¯¯¯¯¯

We now subtract the new multiplied term beneath the immediate one from the immediate one to get;

(3/2)x³ + (11/2)x²

______________________

4x²-12x+9 |6x^(5)+4x⁴-27x³-7x²+27x+3/2

6x^(5)-18x⁴-(27/2)x³

¯¯¯¯¯¯¯¯¯¯¯¯¯¯¯¯¯¯¯¯¯¯¯

22x⁴+(27/2)x³-7x²+27x +3/2

22x⁴-66x³ + (99/2)x²

¯¯¯¯¯¯¯¯¯¯¯¯¯¯¯¯¯¯¯¯¯¯¯

(159/2)x³-(113/2)x²+27x+(3/2)

We'll now divide the term in new polynomial gotten with the highest power by the term with the highest power outside the division symbol. This gives;

(3/2)x³ + (11/2)x² + (159/8)x

______________________

4x²-12x+9 |6x^(5)+4x⁴-27x³-7x²+27x+3/2

6x^(5)-18x⁴-(27/2)x³

¯¯¯¯¯¯¯¯¯¯¯¯¯¯¯¯¯¯¯¯¯¯¯

22x⁴+(27/2)x³-7x²+27x +3/2

22x⁴-66x³ + (99/2)x²

¯¯¯¯¯¯¯¯¯¯¯¯¯¯¯¯¯¯¯¯¯¯¯

(159/2)x³-(113/2)x²+27x+(3/2)

(159/2)x³-(477/2)x²+(1431/8)x

We now subtract the new multiplied term beneath the immediate one from the immediate one to get;

(3/2)x³ + (11/2)x² + (159/8)x

______________________

4x²-12x+9 |6x^(5)+4x⁴-27x³-7x²+27x+3/2

6x^(5)-18x⁴-(27/2)x³

¯¯¯¯¯¯¯¯¯¯¯¯¯¯¯¯¯¯¯¯¯¯¯

22x⁴+(27/2)x³-7x²+27x +3/2

22x⁴-66x³ + (99/2)x²

¯¯¯¯¯¯¯¯¯¯¯¯¯¯¯¯¯¯¯¯¯¯¯

(159/2)x³-(113/2)x²+27x+(3/2)

(159/2)x³-(477/2)x²+(1431/8)x

¯¯¯¯¯¯¯¯¯¯¯¯¯¯¯¯¯¯¯¯¯¯¯¯¯

182x²-(1215/8)x + (3/2)

We'll now divide the term in new polynomial gotten with the highest power by the term with the highest power outside the division symbol. This gives;

(3/2)x³+(11/2)x²+(159/8)x+(91/2)

______________________

4x²-12x+9 |6x^(5)+4x⁴-27x³-7x²+27x+3/2

6x^(5)-18x⁴-(27/2)x³

¯¯¯¯¯¯¯¯¯¯¯¯¯¯¯¯¯¯¯¯¯¯¯

22x⁴+(27/2)x³-7x²+27x +3/2

22x⁴-66x³ + (99/2)x²

¯¯¯¯¯¯¯¯¯¯¯¯¯¯¯¯¯¯¯¯¯¯¯

(159/2)x³-(113/2)x²+27x+(3/2)

(159/2)x³-(477/2)x²+(1431/8)x

¯¯¯¯¯¯¯¯¯¯¯¯¯¯¯¯¯¯¯¯¯¯¯¯¯

182x²-(1215/8)x + (3/2)

182x²-545x + 819/2

We now subtract the new multiplied term beneath the immediate one from the immediate one to get;

182x² - (1215/8)x + (3/2) - 182x² + 545x - 819/2 = (3145/8)x - 408

Remainder = (3145/8)x - 408

Now change matrix B to a 3 x 3 matrix and enter
these values for B:
B=
1.2 1.4 3.1
2.2 1.1 5.6
3.7 4.2 6.7

Answers

The 3×3 Matrix is  [tex]B=\left[\begin{array}{ccc}1.2&1.4&3.1\\2.2&1.1&5.6\\3.7&4.2&6.7\end{array}\right][/tex]

What is Matrix?

In mathematics, a matrix is a rectangular array or table of numbers, symbols, or expressions, arranged in rows and columns, which is used to represent a mathematical object or a property of such an object

What is 3×3 Matrix?

A 3 x 3 matrix is calculated for a matrix having 3 rows and 3 columns

Given,

B=

1.2 1.4 3.1

2.2 1.1 5.6

3.7 4.2 6.7

Then the 3×3 matrix is

[tex]B=\left[\begin{array}{ccc}1.2&1.4&3.1\\2.2&1.1&5.6\\3.7&4.2&6.7\end{array}\right][/tex]

Hence, The 3×3 Matrix is [tex]B=\left[\begin{array}{ccc}1.2&1.4&3.1\\2.2&1.1&5.6\\3.7&4.2&6.7\end{array}\right][/tex]

Learn more about Matrix and 3×3 Matrix here

https://brainly.com/question/12759849

#SPJ2

Answer:

Step-by-step explanation:

c11 =

⇒ 56.1  

c12 =

⇒ 12.1

c13 =

⇒ 236

16p - 32q + 5 when p= 2 and q = 1​

Answers

Answer:

0

Step-by-step explanation:

16x2= 32 and 32x1=32 so 32-32=0

How to solve

1) 16(2) - 32(1) + 5

2) 16 x 2 = 32

3) -32 x 1 = -32

4) 32 - 32 + 5 = 5

Answer is 5

I really help worth these question.

Answers

Answer:

[tex] \frac{7}{3} [/tex]

Step-by-step explanation:

Given that,

p = -6,

q = 6

r = -19

Plug in the above values to evaluate the expression, [tex] \frac{\frac{q}{2} - \frac{r}{3}}{\frac{3p}{6} + \frac{q}{6}} [/tex]

[tex] \frac{\frac{6}{2} - \frac{(-19)}{3}}{\frac{3(-6)}{6} + \frac{6}{6}} [/tex]

[tex] \frac{\frac{3}{1} - \frac{(-19)}{3}}{\frac{-3}{1} + \frac{1}{1}} [/tex]

[tex] \frac{\frac{9 -(-19)}{3}}{3 + 1} [/tex]

[tex] \frac{\frac{28}{3}}{4} [/tex]

[tex] \frac{28}{3}*\frac{1}{4} [/tex]

[tex] \frac{28*1}{3*4} [/tex]

[tex] \frac{7*1}{3*1} [/tex]

[tex] \frac{7}{3} [/tex]

Find the area of the region that lies inside the first curve and outside the second curve.
r= 10cos( θ)
r= 5

Answers

Answer:

Step-by-step explanation:

From the given information:

r = 10 cos( θ)

r = 5

We are to find the  the area of the region that lies inside the first curve and outside the second curve.

The first thing we need to do is to determine the intersection of the points in these two curves.

To do that :

let equate the two parameters together

So;

10 cos( θ) = 5

cos( θ) = [tex]\dfrac{1}{2}[/tex]

[tex]\theta = -\dfrac{\pi}{3}, \ \ \dfrac{\pi}{3}[/tex]

Now, the area of the  region that lies inside the first curve and outside the second curve can be determined by finding the integral . i.e

[tex]A = \dfrac{1}{2} \int \limits^{\dfrac{\pi}{3}}_{-\dfrac{\pi}{3}} (10 \ cos \ \theta)^2 d \theta - \dfrac{1}{2} \int \limits^{\dfrac{\pi}{3}}_{-\dfrac{\pi}{3}} \ \ 5^2 d \theta[/tex]

[tex]A = \dfrac{1}{2} \int \limits^{\dfrac{\pi}{3}}_{-\dfrac{\pi}{3}} 100 \ cos^2 \ \theta d \theta - \dfrac{25}{2} \int \limits^{\dfrac{\pi}{3}}_{-\dfrac{\pi}{3}} \ \ d \theta[/tex]

[tex]A = 50 \int \limits^{\dfrac{\pi}{3}}_{-\dfrac{\pi}{3}} \begin {pmatrix} \dfrac{cos \ 2 \theta +1}{2} \end {pmatrix} \ \ d \theta - \dfrac{25}{2} \begin {bmatrix} \theta \end {bmatrix}^{\dfrac{\pi}{3}}_{-\dfrac{\pi}{3}}[/tex]

[tex]A =\dfrac{ 50}{2} \int \limits^{\dfrac{\pi}{3}}_{-\dfrac{\pi}{3}} \begin {pmatrix} {cos \ 2 \theta +1} \end {pmatrix} \ \ d \theta - \dfrac{25}{2} \begin {bmatrix} \dfrac{\pi}{3} - (- \dfrac{\pi}{3} )\end {bmatrix}[/tex]

[tex]A =25 \begin {bmatrix} \dfrac{sin2 \theta }{2} + \theta \end {bmatrix}^{\dfrac{\pi}{3}}_{\dfrac{\pi}{3}} \ \ - \dfrac{25}{2} \begin {bmatrix} \dfrac{2 \pi}{3} \end {bmatrix}[/tex]

[tex]A =25 \begin {bmatrix} \dfrac{sin (\dfrac{2 \pi}{3} )}{2}+\dfrac{\pi}{3} - \dfrac{ sin (\dfrac{-2\pi}{3}) }{2}-(-\dfrac{\pi}{3}) \end {bmatrix} - \dfrac{25 \pi}{3}[/tex]

[tex]A = 25 \begin{bmatrix} \dfrac{\dfrac{\sqrt{3}}{2} }{2} +\dfrac{\pi}{3} + \dfrac{\dfrac{\sqrt{3}}{2} }{2} + \dfrac{\pi}{3} \end {bmatrix}- \dfrac{ 25 \pi}{3}[/tex]

[tex]A = 25 \begin{bmatrix} \dfrac{\sqrt{3}}{2 } +\dfrac{2 \pi}{3} \end {bmatrix}- \dfrac{ 25 \pi}{3}[/tex]

[tex]A = \dfrac{25 \sqrt{3}}{2 } +\dfrac{25 \pi}{3}[/tex]

The diagrammatic expression showing the area of the region that lies inside the first curve and outside the second curve can be seen in the attached file below.

Find the velocity, acceleration, and speed of a particle with position function r(t)=⟨−8tsint,−8tcost,2t2⟩

Answers

Answer:

The answer is below

Step-by-step explanation:

Velocity is the rate of change of displacement. Velocity is the ratio of distance to time.

The velocity v(t) = [tex]\frac{d}{dt}r(t)[/tex]

Where r(t) is the position function

Given that:

r(t)=⟨−8tsint,−8tcost,2t²⟩

[tex]v(t)=\frac{d}{dt}r(t)= <-8tcost-8sint,8tsint-8cost,4t>[/tex]

Acceleration is the rate of change of velocity, it is the ratio of velocity to time. Acceleration a(t) is given as:

[tex]a(t)=\frac{d}{dt}v(t)= \frac{d}{dt} <-8tcost-8sint,8tsint-8cost,4t>\\=<8tsint-16cost,8tcost+16cost,4>\\\\a(t)=<8tsint-16cost,8tcost+16cost,4>[/tex]

Speed = |v(t)| = [tex]\sqrt{(-8tcost-8sint)^2+(8tsint-8cost)^2+(4t)^2}\\\\ =\sqrt{64t^2cos^2t+128tcostsint+64sin^2t+64t^2sin^2t-128tsintcost+64cos^2t+16t^2}\\ \\=\sqrt{64t^2cos^2t+64t^2sin^2t+64sin^2t+64cos^2t+16t^2}\\\\=\sqrt{64t^2(cos^2t+sin^2t)+64(sin^2t+cos^2t)+16t^2}\\\\=\sqrt{64t^2+64+16t^2}=\sqrt{80t^2+64}[/tex]

Simplify and Show all of your work please!

Answers

Answer:

[tex]\Huge \boxed{\mathrm{9}}[/tex]

Step-by-step explanation:

[tex]\Rightarrow \displaystyle \frac{18 \div 2 * 3}{5-2}[/tex]

Dividing first.

[tex]\Rightarrow \displaystyle \frac{9 * 3}{5-2}[/tex]

Multiplying and subtracting.

[tex]\Rightarrow \displaystyle \frac{27}{3}[/tex]

Division.

[tex]\Rightarrow 9[/tex]

the perimeter of a rectangle is 12cm. If the length is 2 less than 3 times the width, find the length

Answers

Answer:

The length of the rectangle would be 4.

Step-by-step explanation:

We can start by naming the width x.

Therefore, the length would 3x-2.

The perimeter of the rectangle would then be:

2(3x-2+x)

We can set up the given equation

2(3x-2+x)=12

Solve for x.

Divide both sides by 2.

3x-2+x=6

Combine like terms.

4x-2=6

Add 2 to both sides.

4x=8

Divide both sides by 4.

x=2

The width would then be 2.

We can plug that into the expression for length.

3x-2

3(2)-2

6-2

4

The length of the rectangle would be 4.

algebra 2
50 POINTS
HELP

Answers

Answer:

{-3, 2}U{2, 5}

Step-by-step explanation:

For an equation to be negative, it would need to be in a negative range (below the x-axis or the coordinates are negative y-values). Therefore, we can examine this question and see that the graph is negative when the function crosses the x-axis at -3 and it remains negative until you reach 2 on the x-axis.

Therefore, the first set of negative values is (-3, 2).

Secondly, applying the same logic as before, the function decreases at 2 and then touches the x-axis again at 5. Therefore, the second negative value would be (2, 5).

The negative values are {-3, 2}U{2, 5}.

Answer:

{-3, 2}U{2, 5}

Step-by-step explanation:

Which x values is the graph below discontinuous

Answers

For any value of X where there’s a jumb is discontinues. -3,-1,1,3

Answer:

-3,-1,1,3,5

Step-by-step explanation:

Answer I need help !!!!!!!!!!!

Answers

Answer:

Pay for the day = $ 123.25

Step-by-step explanation:

From the question given:

Monday morning:

Time in: 8:15

Time out: 12:15 pm

Monday afternoon:

Time in: 13:00

Time out: 17:30

Pay = $ 14.5 /hr

Next, we shall determine the number of hours of work in the morning. This is illustrated below:

Time in: 8:15

Time out: 12:15 pm

Difference in time = 12:15 – 8:15 = 4 hrs

Next, we shall determine the pay for the work done in the morning. This can be obtained as follow:

Pay = $ 14.5 /hr

Pay for work done in the morning

= 4 × 14.5 = $ 58

Next, we shall determine the number of hours of work in the afternoon. This is illustrated below:

Time in: 13:00

Time out: 17:30 pm

Difference in time = 17:30 – 13:00 = 4 hrs 30 minutes

Next, we shall convert 4 hrs 30 minutes to hours. This is illustrated below:

60 minutes = 1 hr

30 minutes = 30/60 = 0.5 hrs.

Therefore,

4 hrs 30 minutes = 4 + 0.5 = 4.5 hrs

Next, we shall determine the pay for the work done in the afternoon. This can be obtained as follow:

Pay = $ 14.5 /hr

Pay for work done in the afternoon

= 4.5 × 14.5 = $ 65.25

Finally, we shall determine the pay for the day as follow:

Pay for work done in the morning

= $ 58

Pay for work done in the afternoon

= $ 65.25

Pay for the day = pay for morning + pay for afternoon

Pay for the day = $ 58 + $ 65.25

Pay for the day = $ 123.25

Therefore, the pay for the day is

$ 123.25

What is mAngleRST in degrees?

Answers

Answer:

The measure of angle RST = 120°

Step-by-step explanation:

Answer:

The measure of angle RST = 120°

hope this helps

Solve the problem. A variable x has the possible observations shown below. Possible observations of x: -3 -1 0 1 1 2 4 4 5 Find the z-score corresponding to an observed value of x of 2.

Answers

Answer:

The z-score corresponding to an observed value of x of 2 is 0.215.

Step-by-step explanation:

We are given that a variable x has the possible observations shown below;

Possible observations of X: -3, -1, 0, 1, 1, 2, 4, 4, 5.

Firstly, we will find the mean and the standard deviation of X, i.e;

Mean of X, ([tex]\mu[/tex]) = [tex]\frac{\sum X}{n}[/tex]

                       =  [tex]\frac{(-3)+ (-1)+ 0+ 1+ 1+ 2+ 4+ 4+ 5}{9}[/tex]  

                       =  [tex]\frac{13}{9}[/tex]  = 1.44

Standard deviation of X, ([tex]\sigma[/tex]) =  [tex]\sqrt{\frac{\sum (X-\bar X)^{2} }{n-1} }[/tex]

                            =  [tex]\sqrt{\frac{(-3-1.44)^{2}+(-1-1.44)^{2}+......+(4-1.44)^{2}+(5-1.44)^{2} }{9-1} }[/tex]

                            =  2.603

Now, the z-score corresponding to an observed value of x of 2 is given by;

                  z-score =  [tex]\frac{X-\mu}{\sigma}[/tex]

                               =  [tex]\frac{2-1.44}{2.603}[/tex]  = 0.215.        

3 πd=12 π
what does d equal?

Answers

__________

D = 4

__________

i blelieve this is it.

Answer:

d =4

Step-by-step explanation:

[tex]3\pi d = 12 \pi\\\\divide\:both\:sides\:of\:the\:equation\:by\: 3\:\pi\\\frac{3 \pi d}{3\pi} = \frac{12 \pi}{3\pi} \\\\d=4[/tex]

If a wind turbine makes 64 full revolutions every 1 minute, what is its angular speed?

Answers

Answer:

this is wind turbine angular speed

Step-by-step explanation:

given data

angular speed ω = 64 rpm

time = 1 min = 60 seconds

                                                                           

solution

we know that angular speed ω is expess as

ω = [tex]\frac{2\pi }{T}[/tex]    .........................1

ω = 64 × [tex]\frac{2\pi }{T}[/tex]

ω  = 6.70 rad/s

so this is wind turbine angular speed

Number of times the individual changed jobs in the last 5 years is what kind of variable? A. This variable is a continuous numerical variable that is interval-scaled. B. This variable is a discrete numerical variable that is interval-scaled. C. This variable is a categorical variable that is ordinal-scaled. D. This variable is a discrete numerical variable that is ratio-scaled. E. This variable is a continuous numerical variable that is ratio-scaled. F. This variable is a categorical variable that is nominal-scaled.

Answers

Answer: D. This variable is a discrete numerical variable that is ratio-scaled.

Step-by-step explanation:

A Discrete variables are variables which are countable in a finite amount of time. For example, you can count the amount of money in your bank wallet, but same can’t be said for the money you have deposited in eveyones bank account as this is infinite.

So the number of times an individual changes job in a five years period is a perfect example of  a discrete numerical variable that is ratio scaled because it can be counted.

The equation a = 640 s gives the relationship between s square miles and a acres. Pam owns 4.5 square miles of farmland. How many acres does she own? a. 2,880 acres b. 288 acres c. 0.7 acres d. 7.03 acres

Answers

Answer:

A. 2880 acres

Step-by-step explanation:

Formula: a = 640 s

Given information: s = 4.5

--> a = 640 x 4.5 = 2880 (acres)

Find an equation of the line passing through the point (−3,−7) that is perpendicular to the line y= −5x+4

Answers

Answer:

y = 1/5x - 6.4

Step-by-step explanation:

Perpendicular lines have opposite reciprocal slopes, so the slope will be 1/5

Then, plug the slope and the point into the equation y = mx + b to find b

y = mx + b

-7 = 1/5(-3) + b

-7 = -0.6 + b

-6.4 = b

Then, plug this and the slope into the equation

y = 1/5x - 6.4 will be the equation

Ana must take 11.25 mL of Medicine A daily. She must take 5.5 mL of Medicine B daily. How many more ml of Medicine A than Medicine B must she take
daily?

Answers

Answer:

5.75

Step-by-step explanation:

11.25 - 5.5 = 5.75.

So, Ana must take 5.5 more mL of Medicine A than Medicine B.

Hope this helps!

which fraction is less then 7/10

Answers

Answer:

Well, there is a lot of answers to that. Check explanation, please!

Step-by-step explanation:

For example, some basic fractions that are less than 1/7 are:

1/8 (Compare their size!)2/18Or 1/154

You can also set up a number line, and compare the fractions.

Hopefully, this answer helps! :D

Tony ran 1/2 of a mile for 1/4 of an hour. How many miles per hour did he run? A)1.0 B)2.0 C)3.0 D)4.0 E)5.0

Answers

Answer:

2.0

Step-by-step explanation:

Answer:

B. 2.0 miles

Step-by-step explanation:

Tony ran 1/2 of a mile for 1/4 of an hour.

First, to make it easier, change each fraction into decimals:

1/2 = 0.5

1/4 = 0.25

It takes Tony 0.25 hours to run 0.5 miles.

You are solving for 1 hours worth. Multiply 4 to both terms:

0.25 hr x 4 = 1 hr

0.5 miles x 4 = 2.0 miles

B. 2.0 miles is your answer.

~

is 27.14159 rational or irrational

Answers

Answer:

It´s rational

Step-by-step explanation:

27,14159 = 2714159/100000

Rational

Robert has available 400 yards of fencing and wishes to enclose a rectangular area. Express the areaAof the rectangle as a function of the widthwof the rectangle. For what value ofwis the arealargest? What is the maximum area?

Answers

Answer:

A) A = 200w - w²

B) w = 100 yards

C) Max Area = 10000 sq.yards

Step-by-step explanation:

We are told that Robert has available 400 yards of fencing.

A) we want to find the expression of the area in terms of the width "w".

Since width is "w", and perimeter is 400,if we assume that length is l, then we have;

2(l + w) = 400

Divide both sides by 2 gives;

l + w = 200

l = 200 - w

Thus, Area of rectangle can be written as;

A = w(200 - w)

A = 200w - w²

B) To find the value of w for which the area is largest, we will differentiate the expression for the area and equate to zero.

Thus;

dA/dw = 200 - 2w

Equating to zero;

200 - 2w = 0

2w = 200

w = 200/2

w = 100 yards

C) Maximum area will occur at w = 100.

Thus;

A_max = 200(100) - 100(100)

A_max = 10000 sq.yards

Round to the nearest cent.
6. $10.407

Answers

Answer:

the answer is 10.41. If you have a number 5 or more you round the nearest number on the left up 1 if it's 4 or less it stays the same it doesn't go up or down.

$10.41 is the correct answer


Solve M= 2HA + 2HT for H.

Answers

Answer:

  H = M/(2A +2T)

Step-by-step explanation:

Factor out H and divide by its coefficient.

  M = 2HA +2HT

  M = H(2A +2T)

  M/(2A +2T) = H

  H = M/(2A +2T)

Solve for w 98 = 7w Simplify your answer as much as possible.

Answers

Answer:

W = 14

Step-by-step explanation:

7w=98

Divide

w=98/7

Done

w=14

Hope this helps! :)

(pls mark brainliest)

Answer:

w = 14

Step-by-step explanation:

98 = 7w

98/7 = 7w/7

14 = w

Hope this helps.

please help ive been stuck on this for a very long time

Answers

It’s A because 3x+1 is = 3 x 2 = 5

By Thales' theorem:

[tex]\[\begin{array}{l}\frac{{18}}{{2x + 2}} = \frac{{24}}{{3x + 1}}\\18(3x + 1) = 24(2x + 2)\\54x + 18 = 48x + 48\\6x = 30\\x = 5\\A.\end{array}\][/tex]

Use the Trapezoidal Rule and Simpson's Rule to approximate the value of the definite integral for the value of n. Round the answers to four decimal places and compare the results with the exact value definite integral.
∫9 4 √xdx,n=8.

Answers

Answer and Step-by-step explanation: The Trapezoidal and Simpson's Rules are method to approximate a definite integral.

Trapezoidal Rule evaluates the area under the curve (definition of integral) by dividing the total area into trapezoids.

The formula to calculate is given by:

[tex]\int\limits^a_b {f(x)} \, dx = \frac{b-a}{2n}[f(x_{0})+2f(x_{1})+2f(x_{2})+...+2f(x_{n-1})+f(x_{n})][/tex]

The definite integral will be:

[tex]\int\limits^9_4 {\sqrt{x} } \, dx = \frac{9-4}{2.8}[2+2.\sqrt{5} +2.\sqrt{6} +2.\sqrt{7}+2.\sqrt{8}+3][/tex]

[tex]\int\limits^9_4 {\sqrt{x} } \, dx = \frac{5}{16}[25.3193][/tex]

[tex]\int\limits^9_4 {\sqrt{x} } \, dx = 7.9122[/tex]

Simpson's Rule divides the area under the curve into an even interval number of subintervals, each with equal width.

The formula to calculate is:

[tex]\int\limits^a_b {f(x)} \, dx = \frac{b-a}{3n}[f(x_{0})+4f(x_{1})+2f(x_{2})+...+2f(x_{n-2})+4f(x_{n-1})+f(x_{n})][/tex]

The definite integral will be:

[tex]\int\limits^9_4 {\sqrt{x} } \, dx = \frac{9-4}{3.8}[2+4.\sqrt{5} +2.\sqrt{6} +4.\sqrt{7} +4\sqrt{8} +3][/tex]

[tex]\int\limits^9_4 {\sqrt{x} } \, dx = \frac{5}{24}[40.7398][/tex]

[tex]\int\limits^9_4 {\sqrt{x} } \, dx = 8.4875[/tex]

Calculating the definite integral by using the Fundamental Theorem of Calculus:

[tex]\int\limits^9_4 {\sqrt{x} } \, dx = \int\limits^9_4 {x^{\frac{1}{2} }} \, dx[/tex]

[tex]\int\limits^9_4 {\sqrt{x} } \, dx = \frac{2.\sqrt[]{x^{3}} }{3}[/tex]

[tex]\int\limits^9_4 {\sqrt{x} } \, dx = \frac{2.\sqrt[]{9^{3}} }{3}-\frac{2.\sqrt[]{4^{3}} }{3}[/tex]

[tex]\int\limits^9_4 {\sqrt{x} } \, dx = 12.6667[/tex]

Comparing results, note that Simpson's Rule is closer to the exact value, i.e., gives better approximation to the exactly value calculated by the fundamental theorem.

411,500 science notation

Answers

Answer:

the answer is 4.115 x 10^5

Step-by-step explanation:

hope that helps

Other Questions
Which of these statements regarding the computer platform is true?A.computer game titles are more expensive than their console counterparts.B. The computer platform is proprietary.C. Requirements vary depending on the game title.D. computer monitors have a lower resolution than television screens. Taho Earns His Regular Pay Of $11 Per Hour For Up To 40 Hours Of Work Per Week. For Each Hour Over 40 Hours Of Work Per week.For each hour over 40 hours of work per week. Taho earns 1 times his regular pay. How much does Taho earn in a week in which he works 50 hours? a. $550 b. $605 c. $625 d. $750 e. $825 explain the relationship between structure and function Is make appointments a prepositional phrase? A bag has 13 red candies 7 pink candy 12 orange candies what is the possibility that you will choose an orange candy at random PLS HELP THIS IS DUE IN 10 MINUTES Describe at least 3 uses of ultraviolet radiation and discuss whether the benefits outwheigh the risk. When do lunar eclipses occur? Greg and Joyce have an adjustable rate mortgage on their home. What is the key feature of this type of loan? Name the line and plane shown in the diagram. PLZ ANSWER ASAP, DUE IN 1 HOUR 14b + 6 = 6 14b = Subtract 6 from both sides b = Divide both sides by 14 The gasoline consumption of a small car is advertised as 16.3 km/L (1L=1liter). How many miles per gallon is this? One mile is 1.609 km and one gallon is 3.788 L. "A customer has an existing short margin account with credits of $16,000 and a short position in ABC stock worth $12,000. The SMA in the account is $1,000. If the market value of ABC falls to $8,000, the equity is:" Of the type of government described which type of government is most similar to the U.S government? Write a story that ends with the statement: I had never felt so embarrassed in my life x2 + 2x -1 = 0 in English words What is an amendment? Match the following organelle with below mention activities.a. drug detoxification :________ b. catalase :________c. cytochrome P-450 :________d. glycosyl transferases :________e. acid hydrolases :________f. turgor pressure :________g. N-linked glycosylation :________1. Rough endoplasmic 2. Reticulum 3. Peroxisomes 4. Plant vacuole 5. Lysosome 6. Smooth 7. Endoplasmic 8. Reticulum 9. Mitochondria 10. Golgi there are 15 black marbles and 10 white marbles in a bag. If you pick a marble from the bag without looking, what are your chances of picking a white marble? One of the most successful stories of wildlife conservation in India is that of the Indian Rhino, which almost went extinct at the beginning of the 20th century. Today there are about 1700 rhinos in the Kaziranga National Park in Assam!Which of the following was probably a critical step in this conservation plan? A) controlling floods in the Brahmaputra B) growing more grass on the Brahamaputra basinC) controlling mass poaching at Kaziranga D) bringing more tourists to Kaziranga the sum of two squares of two consecutive natural numbers is 41.Find the numbers